Solved papers for BCECE Engineering BCECE Engineering Solved Paper-2013

done BCECE Engineering Solved Paper-2013 Total Questions - 150

  • question_answer1) The dimensions of universal gravitational constant are

    A)
    \[[{{M}^{-2}}{{L}^{-3}}{{T}^{-2}}]\]                          

    B)
     \[[{{M}^{-2}}{{L}^{2}}{{T}^{-1}}]\]          

    C)
     \[[{{M}^{-1}}{{L}^{3}}{{T}^{-2}}]\]          

    D)
           \[[M{{L}^{2}}{{T}^{-1}}]\]

    View Answer play_arrow
  • question_answer2) If\[|A\times B|=\sqrt{3}A.B\] then the value of\[|A+B|\] is

    A)
     \[A+B\]                              

    B)
     \[{{({{A}^{2}}+{{B}^{2}}+\sqrt{3}AB)}^{1/2}}\]

    C)
     \[{{\left( {{A}^{2}}+{{B}^{2}}+\frac{AB}{\sqrt{3}} \right)}^{1/2}}\]

    D)
     \[{{({{A}^{2}}+{{B}^{2}}+AB)}^{1/2}}\]

    View Answer play_arrow
  • question_answer3) Two spheres of masses m and M are situated in air and the gravitional force between them is F. The space around the masses is now filled with a liquid of specific gravity 3.The new gravitational force will be

    A)
     F                     

    B)
           \[\frac{F}{3}\]                   

    C)
     3 F                        

    D)
            \[\frac{F}{9}\]

    View Answer play_arrow
  • question_answer4) A body floats in water with one fourth of its volume above the surface of water. If placed in oil it floats with one third of its volume above the surface of oil. The density of oil is

    A)
     \[\frac{3}{4}\]                  

    B)
            \[\frac{4}{9}\]                  

    C)
     \[\frac{2}{3}\]                  

    D)
            \[\frac{9}{8}\]

    View Answer play_arrow
  • question_answer5) When capillary tubes of different radii  r dipped in water, water rises to different heights K in them, then

    A)
    \[h{{r}^{2}}=\text{constant}\]

    B)
            \[hr=\text{constant}\]

    C)
            \[\frac{h}{r}=\text{constant}\]

    D)
            \[\frac{h}{{{r}^{2}}}=\text{constant}\]

    View Answer play_arrow
  • question_answer6) In the adjoining figure A, B, and C represents three progressive waves. Which of the following statement about the waves is correct?

    A)
     Wave C lags behind in phase by \[\frac{\pi }{2}\]from A and B leads by \[\frac{\pi }{2}\]

    B)
     Wave C leads in phase by \[\pi \]from A and B lags behind by \[\pi \]

    C)
     Wave C leads in phase by \[\frac{\pi }{2}\]from A and lags behind by \[\frac{\pi }{2}\]

    D)
     Wave C lags behind in phase by \[\pi \]from A and \[\pi \]

    View Answer play_arrow
  • question_answer7) If 110 J of heat are added to a gaseous system, whose internal energy is 40J, then the -amount of external work done is

    A)
     40 J       

    B)
            70 J   

    C)
     110 J    

    D)
            150 J

    View Answer play_arrow
  • question_answer8) A ray is incident at an angle of incidence \[i\] on one surface of a prism of small angle A and emerges normally from the opposite surface. If the refractive index of the material of the prism is \[\mu ,\]the angle of incidence \[i\] is nearly equals to

    A)
     \[\mu \,A/2\]

    B)
                            \[\,A/2\mu \]  

    C)
            \[\,\mu A\]       

    D)
                            \[\,A/\mu \]

    View Answer play_arrow
  • question_answer9) Figure below shows four plates each of area S separated from one another by a distance d. What is the capacitance between A and B?

    A)
    \[\frac{4\,{{\varepsilon }_{0}}3}{d}\]                      

    B)
     \[\frac{3\,{{\varepsilon }_{0}}5}{d}\]

    C)
     \[\frac{2\,{{\varepsilon }_{0}}5}{d}\]     

    D)
            \[\frac{\,{{\varepsilon }_{0}}5}{d}\]

    View Answer play_arrow
  • question_answer10) What is the value of current in the arm containing \[2\pi \] resistor in .the adjoining circuit?

    A)
     0.7 amp          

    B)
            1.2 amp

    C)
     1.5 amp          

    D)
            1.0 amp

    View Answer play_arrow
  • question_answer11) For ohmic conductor the drift velocity\[{{\upsilon }_{d}}\] and the electric field applied across it are related as

    A)
    \[{{v}_{d}}\propto \sqrt{E}\]     

    B)
            \[{{v}_{d}}\propto {{E}^{2}}\]

    C)
     \[{{v}_{d}}\propto E\]  

    D)
            \[{{v}_{d}}\propto \frac{1}{E}\]

    View Answer play_arrow
  • question_answer12) If E and B be the electric and magnetic field vectors of electromagnetic waves, the direction of propagation of electromagnetic waves is that of

    A)
     E                            

    B)
            B     

    C)
    \[E\times B\]   

    D)
           \[B\times E\]

    View Answer play_arrow
  • question_answer13) The current in RCL circuit is maximum where

    A)
     \[{{X}_{L}}=0\]                

    B)
     \[{{X}_{L}}={{X}_{C}}\]

    C)
     \[{{X}_{C}}=0\]

    D)
            \[X_{L}^{2}+X_{C}^{2}=1\]

    View Answer play_arrow
  • question_answer14) What is the value of \[\bar{A}+A\]in the Boolean algebra?

    A)
     0            

    B)
            1        

    C)
     A           

    D)
            \[\bar{A}\]

    View Answer play_arrow
  • question_answer15) The truth table given below is for which gate?
    A B Y
    0 0 1
    0 1 1
    1 0 1
    1 1 0

    A)
     XOR                                     

    B)
     OR

    C)
     AND                     

    D)
            NAND

    View Answer play_arrow
  • question_answer16) The magnetic flux through a circuit of resistance R changes by an amount \[\Delta \phi \] in a time \[\Delta t.\]Then the total quantity of electric charge Q that passes any point in the circuit during the time \[\Delta t\]is represented by

    A)
     \[Q=\frac{\Delta \phi }{\Delta t}\]          

    B)
            \[Q=\frac{\Delta \phi }{R}\]       

    C)
            \[Q=R.\frac{\Delta \phi }{\Delta t}\]      

    D)
            \[Q=\frac{1}{R}\frac{\Delta \phi }{\Delta t}\]

    View Answer play_arrow
  • question_answer17) A particle moves along a circle of radius \[\left( \frac{20}{\pi } \right)\,m\]with constant tangential acceleration. If the velocity of the particle is 80 m/s at the end of the second revolution after motion has begun the tangential acceleration will be

    A)
    \[40\pi \text{ }m/{{s}^{2}}\]         

    B)
          \[~40\text{ }m/{{s}^{2}}\]           

    C)
           \[~160\pi \,m/{{s}^{2}}\]       

    D)
            \[~640\pi \,m/{{s}^{2}}\]

    View Answer play_arrow
  • question_answer18) The output of a OR gate is 1

    A)
     if either input is zero

    B)
     if both inputs are zero

    C)
     only if both inputs are 1

    D)
     if either or both inputs are 1

    View Answer play_arrow
  • question_answer19)  Two 220 V, 100 W bulbs are connected first in series and then in parallel. Each time the combination is connected to a 220V AC supply line, the power drawn by the combination in each case respectively will be

    A)
    \[50,200W\]     

    B)
          \[~50W,20W\text{ }\]   

    C)
           \[100\text{ }W,50\text{ }W\]   

    D)
           \[~200\text{ }W,150\text{ }W\]

    View Answer play_arrow
  • question_answer20) Barrier potential of a p-n junction diode does not depend on

    A)
     diode design      

    B)
           doping density

    C)
     temperature      

    D)
                            farward bias

    View Answer play_arrow
  • question_answer21) A projectile is projected with a linear momentum p making an angle \[\theta \] with the horizontal. The change in momentum of the projectile on return to the ground will be

    A)
    \[2p\,tan\,\theta \]          

    B)
            \[2p\,sin\,\theta \]       

    C)
           \[2p\,cos\,\theta \]         

    D)
           \[~2p\]

    View Answer play_arrow
  • question_answer22) A weigthtless thread can bear tension up to  \[\text{3}\text{.7 kg wt}\text{.}\] A stone of mass 500 g is tied to it and revolved in a circular path of radius 4m in a vertical plane. If \[g=10\text{ }m/{{s}^{2}},\]then the maximum angular velocity of the stone will be

    A)
     2 rad/s              

    B)
            4 rad/s

    C)
            16 rad/s             

    D)
           \[\sqrt{21}\,rad/s\]

    View Answer play_arrow
  • question_answer23) Under a constant torque, the angular momentum of a body changes from A to 4 A in 4 s. The torque is

    A)
     3 A        

    B)
           \[\frac{1}{4}A\]                

    C)
           \[\frac{3}{4}A\]                

    D)
            \[\frac{4}{3}A\]

    View Answer play_arrow
  • question_answer24) The mass of a planet is double and its radius is half compared to that of earth. Assuming \[g=10m/{{s}^{2}}\]on earth, the acceleration due to gravity at the planet will be

    A)
    \[10\text{ }m/{{s}^{2}}\]            

    B)
            \[20\text{ }m/{{s}^{2}}\]            

    C)
           \[~40\text{ }m/{{s}^{2}}\]            

    D)
            None of these

    View Answer play_arrow
  • question_answer25) A soap bubble in vacuum, has a radius of 3 cm and another soap bubble in vacuum has a radius of 4 cm. If the two bubbles coalesce under isothermal condition, then the radius of new bubble is

    A)
     4.3 cm                 

    B)
            4.5 cm                 

    C)
     5 cm                     

    D)
            7 cm

    View Answer play_arrow
  • question_answer26) Deuteron and \[\alpha -\]particle are put \[\text{1}\overset{\text{o}}{\mathop{\text{A}}}\,\]apart in air. Magnitude of intensity of electric field due to deuteron of \[\alpha -\]particle is

    A)
     zero                     

    B)
            \[2.88\times {{10}^{11}}N/C\]  

    C)
     \[1.44\times {{10}^{11}}N/C\]

    D)
            \[5.76\times {{10}^{11}}N/C\]

    View Answer play_arrow
  • question_answer27) The air column in pipe, which is closed at one end will be in resonance with a vibrating turning fork at a frequency of 260 Hz, if the length of the air column is

    A)
     31.73 cm          

    B)
            62.5 cm               

    C)
            35.75 cm          

    D)
            12.5 cm

    View Answer play_arrow
  • question_answer28) A ball strikes against the floor and returns with double the velocity. In which type of collision is it possible?

    A)
     Inelastic

    B)
     Perfectly inelastic

    C)
     Perfectly elastic

    D)
     Not possible

    View Answer play_arrow
  • question_answer29) A body is under the action of three force \[{{F}_{1}},{{F}_{2}},\] and \[{{F}_{3}}.\]In which case the body cannot under go angular acceleration?

    A)
     \[{{F}_{1}}+{{F}_{2}}+{{F}_{3}}=0\]

    B)
     \[{{F}_{1}},{{F}_{2}}\]and\[{{F}_{3}}\]are concurrent

    C)
     \[{{F}_{1}},\]and\[{{F}_{2}}\]act at the same point but, \[{{F}_{3}}\] acts at different point

    D)
    \[{{F}_{1}}+{{F}_{2}}\] is parallel to \[{{F}_{3}},\]but the three forces are not concurrent

    View Answer play_arrow
  • question_answer30) Two beams of protons moving parallel in same direction will

    A)
     repel each other

    B)
     exert no force

    C)
     attract each other

    D)
     deflect perpendicular to the plane of the beams

    View Answer play_arrow
  • question_answer31) A photon and an electron possess same de-Broglie wavelength given that \[C=\]speed of light and \[\upsilon =\]space of electron, which of the following relation is correct? (here, \[{{E}_{e}}=K.E\]of electron,\[{{E}_{Ph}}=K.E\] of photon, \[{{P}_{e}}=\] momentum of electron, \[{{P}_{ph}}=\]momentum of photon)

    A)
     \[\frac{{{P}_{e}}}{{{P}_{Pn}}}=\frac{C}{2v}\]     

    B)
            \[\frac{{{E}_{e}}}{{{E}_{Ph}}}=\frac{C}{2v}\]      

    C)
    \[\frac{{{E}_{ph}}}{{{E}_{e}}}=\frac{2c}{v}\]        

    D)
            \[\frac{{{P}_{e}}}{{{P}_{Ph}}}=\frac{2C}{v}\]

    View Answer play_arrow
  • question_answer32) To get an output Y = 1 from circuit of adjoining figure, the input must be

    A)
     A-0        B-1         C-0

    B)
            A-1        B-0         C-0

    C)
            A-1        B-0         C-1

    D)
            A-1        B-1         C-0

    View Answer play_arrow
  • question_answer33) Hubbles law is expressed as (here, \[\upsilon =\]speed of recession, r = distance of galaxy, H = Hubble constant)

    A)
     \[v=Hr\]

    B)
            \[v={{H}^{2}}r\]              

    C)
            \[v=\frac{H}{{{r}^{2}}}\]                              

    D)
            \[v=H{{r}^{2}}\]

    View Answer play_arrow
  • question_answer34) The displacement versus time graph for a body moving in a straight line is shown in figure. Which of the following regions represents motor when no force is acting on the body?

    A)
     ab                         

    B)
            be                         

    C)
            cd                                         

    D)
            be

    View Answer play_arrow
  • question_answer35) Which of the following graphs shows the variation of magnetic field B, with distance from a long current carrying conductor?

    A)
                                       

    B)
          

    C)
                                                               

    D)
          

    View Answer play_arrow
  • question_answer36) The time period of a freely suspended magnet does not depend upon

    A)
     length of the magnet

    B)
            the pole strength of the magnet

    C)
            the horizontal component of magnetic field of earth

    D)
            the length of the suspension

    View Answer play_arrow
  • question_answer37) The magnetic permeability is denned as the ratio of

    A)
     magnetic induction and magnetizing field

    B)
            intensity of magnetization and magnetizing field

    C)
            intensity of magnetization and magnetic field

    D)
            None of the above

    View Answer play_arrow
  • question_answer38) Figure represents an area \[A=0.5\,{{m}^{2}}\]situated in a uniform magnetic field \[B=2.0\,Wb/{{m}^{2}}\]and making an angle of \[60{}^\circ \] with respect to magnetic field. The value of magnetic flux through the area will be

    A)
    \[0.5\,Wb\]                      

    B)
    \[\sqrt{3}\,Wb\]                             

    C)
    \[\frac{3}{2}Wb\]                           

    D)
    \[2.0\,Wb\]

    View Answer play_arrow
  • question_answer39) Of the given diodes, shown in the adjoining diagrams, which one is reverse biased?

    A)
     

    B)
            

    C)
            

    D)
            

    View Answer play_arrow
  • question_answer40) Three particles each of mass m gram, are situated at the vertices of an equilateral triangle ABC of the side 1 cm. The moment of inertia of the system (as shown in figure) about a line AX perpendicular to AB and in the plane of ABC in gram- cm2 units will be

    A)
     \[\frac{3}{2}m{{l}^{2}}\]                              

    B)
            \[\frac{3}{4}m{{l}^{2}}\]                              

    C)
            \[2\,m{{l}^{2}}\]                             

    D)
            \[\frac{5}{4}m{{l}^{2}}\]

    View Answer play_arrow
  • question_answer41) A galvanometer of \[50\pi \] resistance has 25 divisions. A current of \[4\times {{10}^{-4}}A\]gives a deflection of one division. To convert this galvanometer into a voltmeter having a range of 25 volt, it should be connected with a resistance of

    A)
     \[245\pi \]as shunt      

    B)
            \[2450\pi \]as series     

    C)
            \[2500\pi \]as shunt     

    D)
            \[2550\pi \]in series

    View Answer play_arrow
  • question_answer42) A car is moving towards a high cliff. The car driver sounds a horn of frequency \[f.\]The reflected sound heard by the driver has the frequency \[2f.\] If \[\upsilon \]be the velocity of sound, then the velocity of the car, in the same velocity units will be

    A)
     \[\frac{v}{\sqrt{2}}\]                                    

    B)
            \[\frac{v}{2}\]                  

    C)
            \[\frac{v}{3}\]                                  

    D)
            \[\frac{v}{4}\]

    View Answer play_arrow
  • question_answer43) A plane glass slab is kept over various coloured letters, the letter which appears least raised is

    A)
     violet                   

    B)
            blue                     

    C)
            green             

    D)
            red

    View Answer play_arrow
  • question_answer44) If the angle of incidence is \[i\]and that of refraction is r. Then the speed of light in the medium to which the light is refracted from air is

    A)
     \[v=C\frac{\sin i}{\cos r}\]                          

    B)
            \[v=C\frac{\cos r}{\cos \,i}\]                     

    C)
            \[v=C\frac{\sin r}{\sin \,i}\]                       

    D)
            \[v=C\frac{\sin i}{\sin \,r}\]

    View Answer play_arrow
  • question_answer45) An electron moves with a velocity v in an electric field E. If the angle between v and E is neither 0 nor \[\pi ,\]the path followed by the electron is

    A)
     parabola                

    B)
            circle                    

    C)
            straight line                 

    D)
            ellipse                                                    

    View Answer play_arrow
  • question_answer46) The internal resistance of a cell of emf 2 V is \[0.1\,\pi .\]If it is connected to a resistance of \[3.9\pi ,\] then the voltage across the cell will be (in volts)

    A)
     2 V                                        

    B)
            0.5 V                    

    C)
            1.95 V                  

    D)
            2.5 V

    View Answer play_arrow
  • question_answer47) An electric kettle takes 4 A current a t220 V. How much time will it take to boil 1 kg of water from room temperature \[\text{20}{{\,}^{\text{o}}}\text{C?}\] (the temperature of boiling water is\[\text{100}{{\,}^{o}}\text{C}\])

    A)
     12.6 min                             

    B)
            12.8 min                             

    C)
            6.3 min               

    D)
            6.4 min

    View Answer play_arrow
  • question_answer48) The phenomenon of pair production is

    A)
     ejection of an electron from a nucleus

    B)
            ejection of an electron from a metal surface

    C)
            ionization of a neutral atom

    D)
            the production of an electron and a positron from \[\gamma -\]rays.

    View Answer play_arrow
  • question_answer49) In an electron microscope if the potential is increased from 20kV to 80 kV, the resolving power R of the microscope will become

    A)
     \[\frac{R}{2}\]                                 

    B)
            \[2R\]                  

    C)
            \[4R\]                                  

    D)
            \[5R\]

    View Answer play_arrow
  • question_answer50) Force acting upon a charged particle kept between the plates of changed capacitor is F. If one of the plates of the capacitor is removed force acting on the same particle will become

    A)
     0                                            

    B)
            F                            

    C)
            \[\frac{F}{2}\]                                  

    D)
            \[2F\]

    View Answer play_arrow
  • question_answer51) \[C{{H}_{3}}-\underset{C{{H}_{3}}}{\overset{C{{H}_{3}}}{\mathop{\underset{|}{\overset{|}{\mathop{C}}}\,}}}\,-\overset{OH}{\mathop{\overset{|}{\mathop{C}}\,}}\,HC{{H}_{3}}\xrightarrow{HBr}A(Predominant)\]Identify A.

    A)
    \[{{(C{{H}_{3}})}_{2}}C(Br)CH{{(C{{H}_{3}})}_{2}}\]

    B)
    \[{{(C{{H}_{3}})}_{3}}CCH(Br)\,C{{H}_{3}}\]

    C)
    \[{{(C{{H}_{3}})}_{3}}COHBrC{{H}_{3}}\]

    D)
    None of the above

    View Answer play_arrow
  • question_answer52) A \[\beta -\]hydroxy carbonyl compound Is obtained by the action of NaOH on

    A)
    \[{{C}_{6}}{{H}_{5}}CHO\]           

    B)
           \[HCHO\]            

    C)
    \[C{{H}_{3}}CHO\]          

    D)
           \[{{(C{{H}_{3}})}_{3}}C.CHO\]

    View Answer play_arrow
  • question_answer53) Which of the following statements is correct?

    A)
    \[+\,l\]effect stabilizes a carbanion

    B)
    \[+\,\,l\] effect stabilizes a carbocation

    C)
    \[-\,\,l\] effect stabilizes a carbanion

    D)
    \[-\,\,l\]effect stabilizes a carbocation

    View Answer play_arrow
  • question_answer54) \[{{C}_{4}}{{H}_{6}}{{O}_{4}}A\xrightarrow{\Delta }{{C}_{3}}{{H}_{6}}{{O}_{2}}B\xrightarrow[\Delta ]{soda\lim e}C\] Compound C is a hydrocarbon, occupying approx 0.75 L volume per gram. Identify A and B.

    A)
    Tartaric acid, propanoic acid

    B)
    Succinic acid, succinic anhydride

    C)
    Maleic anhydride, maleic acid

    D)
    Methyl malonic acid, propanoic acid

    View Answer play_arrow
  • question_answer55) What is effective nuclear charge and the periphery of nitrogen atom when an extra electron is added in the formation of an anion?

    A)
    1.20                 

    B)
           2.45                       

    C)
    3.55                

    D)
    5.95

    View Answer play_arrow
  • question_answer56) A certain metal sulphide, \[\text{M}{{\text{S}}_{\text{2}}}\text{,}\]is used extensively as a high temperature lubricant. If \[\text{M}{{\text{S}}_{\text{2}}}\]has 40.06 % sulphur by weight, atomic mass of At will be

    A)
    100 amu         

    B)
           96 amu

    C)
           60 amu          

    D)
           30 amu

    View Answer play_arrow
  • question_answer57) In HNC, which element has least value of formal charge?

    A)
    H                            

    B)
           N

    C)
    C                            

    D)
                           All have same value

    View Answer play_arrow
  • question_answer58) Consider following unit values of energy
    (i) 1 L atm,
    (ii) 1 erg,
    (iii) 1 J
    (iv) kcal,
    Increasing order of these values is

    A)
    I = II = Ill = IV                     

    B)
    I < II < III < IV     

    C)
           II < III < I < IV      

    D)
           IV < l < III < II

    View Answer play_arrow
  • question_answer59) The number of elements in the transition metal series Sc through Zn that have four unpaired electrons in their + 2 state are

    A)
    2                     

    B)
           4                             

    C)
    5                      

    D)
    6

    View Answer play_arrow
  • question_answer60) In an atmosphere with industrial smog, copper corrodes to
    (i)\[C{{u}_{2}}{{(OH)}_{2}}S{{O}_{4}}\]
    (ii) \[C{{u}_{2}}{{(OH)}_{2}}C{{O}_{3}}\]
    (iii) \[CuS{{O}_{4}}\]
    (iv) \[CuC{{O}_{3}}\]

    A)
    I and III                

    B)
           II and IV              

    C)
           I and IV

    D)
           l and II

    View Answer play_arrow
  • question_answer61) Mischmetal is

    A)
    an alloy of lanthanide and nickel

    B)
    an alloy of lanthanide and copper

    C)
    an alloy of lanthanide, iron and carbon

    D)
    an alloy of lanthanide, magnesium and nickel

    View Answer play_arrow
  • question_answer62) What is the oxidation state of iron in \[{{[Fe{{({{H}_{2}}O)}_{5}}NO]}^{2+}}\]?

    A)
    0                      

    B)
           + 1                         

    C)
    + 2                   

    D)
    + 3

    View Answer play_arrow
  • question_answer63) Identify the final product of the reaction \[BC{{l}_{3}}+N{{H}_{4}}Cl\xrightarrow[{{C}_{6}}{{H}_{5}}Cl]{140{{\,}^{o}}C}\xrightarrow{NaB{{H}_{4}}}\]?

    A)
    \[{{B}_{3}}{{N}_{3}}{{H}_{6}}\]

    B)
                           \[{{B}_{2}}{{H}_{6}}\]                    

    C)
    \[NaBC{{l}_{4}}\]             

    D)
           BN

    View Answer play_arrow
  • question_answer64) 2.56 g of sulphur in 100 g of \[\text{C}{{\text{S}}_{\text{2}}}\]has depression in freezing point of \[0.010{{\,}^{o}}C.\] Atomicity of sulphur in \[C{{S}_{2}}\]is (Given,\[K{{}_{f}}={{0.1}^{o}}\,\text{mola}{{\text{l}}^{-1}}\])

    A)
    2            

    B)
                           4                             

    C)
    6                     

    D)
    8

    View Answer play_arrow
  • question_answer65) For the zeroth order reaction, sets I and II are given, hence \[x\] is
    I.
    II.

    A)
    2 min                

    B)
           4 min    

    C)
           6 min                

    D)
           8 min

    View Answer play_arrow
  • question_answer66) Equilibrium constant for the reaction, \[N{{H}_{4}}OH+{{H}^{+}}NH_{4}^{+}+{{H}_{2}}O\]is  \[1.8\times {{10}^{9}}.\] Hence,    equilibrium   constant   for\[N{{H}_{3}}(aq)+{{H}_{2}}ONH_{4}^{+}+O{{H}^{-}}\]is

    A)
    \[1.8\times {{10}^{-5}}\]              

    B)
           \[1.8\times {{10}^{5}}\]

    C)
           \[1.8\times {{10}^{-9}}\]              

    D)
           \[5.59\times {{10}^{-10}}\]

    View Answer play_arrow
  • question_answer67) Calculate pH change when 0.01 mol \[\text{C}{{\text{H}}_{\text{3}}}\text{KOO}\,\text{Na}\]solution is added to 1L of \[\text{0}\text{.01}\,\text{M}\,\text{C}{{\text{H}}_{\text{3}}}\text{COOH}\]solution. \[{{K}_{a}}(C{{H}_{3}}COOH)=1.8\times {{10}^{-5}},p{{K}_{a}}=4.74\]

    A)
    3.37                 

    B)
           1.37                       

    C)
    4.74                

    D)
    8.01

    View Answer play_arrow
  • question_answer68) \[p{{K}_{b}}\]of \[\text{N}{{\text{H}}_{\text{3}}}\]is 4.74 and \[\text{p}{{\text{K}}_{b}}\]of \[{{A}^{-}},{{B}^{-}}\]and \[{{C}^{-}}\] are 4,5 and 6 respectively. Aqueous solution of 0.01 M has pH in the increasing order

    A)
    \[N{{H}_{4}}A<N{{H}_{4}}B<N{{H}_{4}}C\]

    B)
    \[N{{H}_{4}}C<N{{H}_{4}}B<N{{H}_{4}}A\]

    C)
    \[N{{H}_{4}}C<N{{H}_{4}}A<N{{H}_{4}}B\]

    D)
    All have equal pH

    View Answer play_arrow
  • question_answer69) Osmotic pressure of insulin solution at 298 K is found to be 0.0072 atm. Hence, height of water column due to this pressure is [given\[d(Hg)=13.6g/mL\]]

    A)
    0.76cm          

    B)
           0.70 cm

    C)
           7.4 cm           

    D)
           76 cm

    View Answer play_arrow
  • question_answer70) \[\Delta {{G}^{o}}\]and \[\Delta {{H}^{o}}\]for a reaction at 300 K is \[-66.9\,\text{kJ}\,\text{mo}{{\text{l}}^{-1}}\] and\[-41.8\,\text{kJ}\,\text{mo}{{\text{l}}^{-1}}\] respectively. \[\Delta {{G}^{o}}\]for the same reaction at 330 K is

    A)
    \[-25.1\,\text{kJ}\,\text{mo}{{\text{l}}^{-1}}\]

    B)
           \[+\,25.1\,\text{kJ}\,\text{mo}{{\text{l}}^{-1}}\]

    C)
           \[18.7\,\text{kJ}\,\text{mo}{{\text{l}}^{-1}}\]

    D)
           \[-69.4\,\text{kJ}\,\text{mo}{{\text{l}}^{-1}}\]

    View Answer play_arrow
  • question_answer71) At 1000 K, from the data \[{{N}_{2}}(g)+3{{H}_{2}}(g)\xrightarrow{{}}2N{{H}_{3}}(g);\] \[\Delta H=-123.77\,\text{kJ}\,\text{mo}{{\text{l}}^{-1}}\]
    Substance \[{{N}_{2}}\] \[{{H}_{2}}\] \[N{{H}_{3}}\]
    P/R 3.5 3.5 4
    Calculate the heat of formation of \[\text{N}{{\text{H}}_{\text{3}}}\text{.}\]at 300 K.

    A)
    \[-\text{ }44.42\text{ kJ mo}{{\text{l}}^{-1}}\]

    B)
    \[-\text{ }88.85\text{ kJ mo}{{\text{l}}^{-1}}\]

    C)
    \[+\text{ }44.42\text{ kJ mo}{{\text{l}}^{-1}}\]

    D)
    \[+\text{ }88.85\text{ kJ mo}{{\text{l}}^{-1}}\]

    View Answer play_arrow
  • question_answer72) Match the electrode (in column I) with its general name (in Column II) and choose the correct option given below.
    Column I Column II
    A. Calomel l. Reference
    B. Glass ll. Redox
    C. Hydrogen Ill. Membrane
    D. Quinhydrone IV. Gas

    A)
    A-III        B-III       C-IV       D-IV

    B)
    A-IIII       B-IV      C-II         D-II

    C)
    A-III        B-II        C-IV        D-I

    D)
    A-II         B-IV       C-III        D-I

    View Answer play_arrow
  • question_answer73) In a Daniell cell constructed in the laboratory, the voltage observed was 0.9 V instead of 1.1 V of the standard cell. A possible explanation is

    A)
    \[[Z{{n}^{2+}}]>[C{{u}^{2+}}]\]

    B)
           \[[Z{{n}^{2+}}]<[C{{u}^{2+}}]\]

    C)
    \[Zn\]electrode has twice the surface of Cu electrode

    D)
    mol ratio of\[~Z{{n}^{2+}}:\text{ }C{{u}^{2+}}\]is 2 : 1

    View Answer play_arrow
  • question_answer74) A quantity of electrical charge that brings about the deposition of 4.5 g Al from \[\text{A}{{\text{l}}^{\text{3+}}}\]at the cathode will also produce the following volume 3t (STP) of \[{{\text{H}}_{2}}(g)\] from \[{{H}^{+}}\]at the cathode

    A)
    44.8 L              

    B)
           22.4 L    

    C)
           11.2 L              

    D)
           5.6 L

    View Answer play_arrow
  • question_answer75) IUPAC name of the following compound is

    A)
    3-propyl cyclo [3, 6] octane

    B)
    1-ethyl tricyclo [2, 3, 0] heptane

    C)
    1-ethyl bicyclo [2, 2,1] heptane

    D)
    4-ethyl bicyclo [2, 2, 2] heptane

    View Answer play_arrow
  • question_answer76) Identify Tin the following series of equation \[H+B{{r}_{2}}\xrightarrow{h\upsilon }\,X\,\xrightarrow[{{D}_{2}}O]{Mg/ether}Y\]

    A)
      \[\overset{Br}{\mathop{}}\,D\]

    B)
    \[\overset{{}}{\mathop{}}\,D\]

    C)
           \[Br\overset{{}}{\mathop{}}\,D\]            

    D)
           \[\underset{D}{\overset{Br}{\mathop{}}}\,\]

    View Answer play_arrow
  • question_answer77) Arrange the following in the order of rate of oxidation with periodic acid
    (i)\[HOC{{H}_{2}}C{{H}_{2}}OH,\]
    (ii) \[C{{H}_{3}}CHOHCHOHC{{H}_{3}}\]
    (iii) \[{{(C{{H}_{3}})}_{2}}COHCOH{{(C{{H}_{3}})}_{2}}\]

    A)
    I > II > III

    B)
                 II > I > III              

    C)
           III > II > I              

    D)
      I > III = II

    View Answer play_arrow
  • question_answer78) Phenol and \[\text{N}{{\text{H}}_{\text{3}}}\]reacts in presence of \[\text{ZnC}{{\text{l}}_{\text{2}}}\] at \[\text{300}{{\,}^{\text{o}}}\text{C}\]to produce

    A)
    tertiary amine   

    B)
           secondary amine

    C)
           primary amine   

    D)
           All of these

    View Answer play_arrow
  • question_answer79) \[A[{{C}_{6}}{{H}_{10}}{{O}_{3}}(keto\,ester)]\xrightarrow[\Delta ]{NaOH+{{I}_{2}}}\]yellow\[ppt+B\xrightarrow{{{H}^{+}}}C\xrightarrow[-C{{O}_{2}}]{\Delta }C{{H}_{3}}COOH.\]Hence, A is

    A)
    \[\text{C}{{\text{H}}_{\text{3}}}\overset{\text{O}}{\mathop{\overset{\text{ }\!\!|\!\!\text{ }\,\text{ }\!\!|\!\!\text{ }}{\mathop{\text{C}}}\,}}\,\text{C}{{\text{H}}_{\text{2}}}\overset{\text{O}}{\mathop{\overset{\text{ }\!\!|\!\!\text{ }\,\text{ }\!\!|\!\!\text{ }}{\mathop{\text{C}}}\,}}\,\text{O}{{\text{C}}_{\text{2}}}{{\text{H}}_{\text{5}}}\]

    B)
           \[\text{C}{{\text{H}}_{\text{3}}}\text{C}{{\text{H}}_{\text{2}}}\overset{\text{O}}{\mathop{\overset{\text{ }\!\!|\!\!\text{ }\,\text{ }\!\!|\!\!\text{ }}{\mathop{\text{C}}}\,}}\,\text{C}{{\text{H}}_{\text{2}}}\overset{\text{O}}{\mathop{\overset{\text{ }\!\!|\!\!\text{  }\!\!|\!\!\text{ }}{\mathop{\text{C}}}\,}}\,\text{OC}{{\text{H}}_{\text{3}}}\]

    C)
    Both are correct 

    D)
       None is correct

    View Answer play_arrow
  • question_answer80) \[\text{C}{{\text{H}}_{\text{3}}}\overset{\text{O}}{\mathop{\overset{\text{ }\!\!|\!\!\text{  }\!\!|\!\!\text{ }}{\mathop{\text{C}}}\,}}\,\text{C}{{\text{H}}_{\text{3}}}\]can be converted into \[\text{C}{{\text{H}}_{\text{3}}}\overset{\text{O}}{\mathop{\overset{\text{ }\!\!|\!\!\text{  }\!\!|\!\!\text{ }}{\mathop{\text{C}}}\,}}\,\text{OH}\]by following methods
    I. \[C{{H}_{3}}\overset{O}{\mathop{\overset{|\,|}{\mathop{C}}\,}}\,C{{H}_{3}}\xrightarrow[\Delta .{{H }^{+}}]{{{I}_{2}}/NaOH}\]
    II. \[C{{H}_{3}}\overset{O}{\mathop{\overset{|\,|}{\mathop{C}}\,}}\,C{{H}_{3}}\xrightarrow{C{{r}_{2}}O_{7}^{2-}/{{H}^{+}}}\]
    III. \[C{{H}_{3}}\overset{O}{\mathop{\overset{|\,|}{\mathop{C}}\,}}\,C{{H}_{3}}\xrightarrow{Ag(N{{H}_{3}})_{2}^{+}}\]
    Which method are most effective?

    A)
    I, III                     

    B)
    II, III                      

    C)
    I, II                  

    D)
    I, II, III

    View Answer play_arrow
  • question_answer81) Relative  stabilities  of the  following carbocation will be in order
    I   
    II
    III   
    IV \[C{{H}_{3}}\overset{\oplus }{\mathop{C}}\,{{H}_{2}}\]

    A)
    IV < III < II < I    

    B)
           IV < II < III < I     

    C)
           II < IV < III < I    

    D)
           I < II < III < IV

    View Answer play_arrow
  • question_answer82) The correct statement in respect of protein haemoglobin is that it

    A)
    functions as a catalyst for biological reactions

    B)
    maintains blood sugar level

    C)
    acts as an oxygen carrier in the blood

    D)
    forms antibodies and offers resistance to diseases

    View Answer play_arrow
  • question_answer83) \[C{{H}_{3}}COOH+HCOOH\xrightarrow[570\,K]{MnO}\] Main product of above reaction is

    A)
    \[C{{H}_{3}}CHO\]                          

    B)
    \[C{{H}_{3}}COC{{H}_{3}}\]

    C)
           \[HCHO\]            

    D)
           \[{{(C{{H}_{3}}CO)}_{2}}O\]

    View Answer play_arrow
  • question_answer84) Aniline is reacted with bromine water and the resulting product is treated with an aqueous solution of sodium nitrite in presence of dilute hydrochloric acid. The compound so formed is converted to a tetrafluoroborate which is subsequently heated. The final product is

    A)
    p-brpmoaniline

    B)
    p-bromofluorobenzene

    C)
    1, 3, 5-tri bromobenzene

    D)
    2, 4, 6-tribromofluorobenzene

    View Answer play_arrow
  • question_answer85) With hard water, ordinary soap forms curdy precipitate of

    A)
    \[{{(RCOO)}_{2}}Ca\]

    B)
           \[{{(RCOO)}_{2}}Mg\]

    C)
           Both (a) and (b) 

    D)
           None of these

    View Answer play_arrow
  • question_answer86) Which is matched incorrectly? Structure                             Compound

    A)
                      Ascorbic acid

    B)
     \[{{(RCOO)}_{2}}Mg\]Coumarin

    C)
    Both (a) and (b)

    D)
    None of the above

    View Answer play_arrow
  • question_answer87) Which base is normally found in RNA but not in DNA?

    A)
    Uracil                    

    B)
    Thymine             

    C)
           Guanine              

    D)
           Adenine

    View Answer play_arrow
  • question_answer88) The dipole moment of HBr is \[2.60\times {{10}^{-30}}\,\text{cm}\]and the interatomic spacing is \[1.41\,\overset{\text{o}}{\mathop{\text{A}}}\,.\] What is the per cent ionic character of HBr?

    A)
    50%                       

    B)
    11.5%   

    C)
           4.01%  

    D)
           1.19%

    View Answer play_arrow
  • question_answer89) An element is oxidized by fluorine and not by chlorine. Identify the element.

    A)
    Sodium                

    B)
    Aluminium         

    C)
           Oxygen            

    D)
           Sulphur

    View Answer play_arrow
  • question_answer90) The reduction of an oxide by aluminium is called

    A)
    Krolls process

    B)
    van Arkel process

    C)
    Ellingham process

    D)
    Goldschmidts aluminothermite process

    View Answer play_arrow
  • question_answer91) \[{{\text{H}}_{\text{2}}}\]gas is liberated at cathode and anode both by electrolysis of the following solution except in

    A)
    NaCI                     

    B)
           NaH                      

    C)
    LiH                         

    D)
           HCOONa

    View Answer play_arrow
  • question_answer92) Identify (A) and (B) in the following sequence of reactions. \[SnC{{l}_{2}}+2NaOH\xrightarrow{{}}(A)(white\,ppt)\]\[\xrightarrow[(excess)]{NaOH}(B)\]

    A)
    \[Sn{{(OH)}_{2}},N{{a}_{2}}Sn{{O}_{3}}\]

    B)
    \[Sn{{(OH)}_{2}},N{{a}_{2}}Sn{{O}_{2}}\]

    C)
    \[Sn{{(OH)}_{2}},N{{a}_{2}}[Sn{{(OH)}_{6}}]\]

    D)
    \[Sn{{(OH)}_{2}},\]no effect

    View Answer play_arrow
  • question_answer93) Nature of nitride ion is

    A)
    acidic              

    B)
           basic     

    C)
           amphiprotic       

    D)
           cannot predict

    View Answer play_arrow
  • question_answer94) Wave number of spectral line for a given transition is \[x\,c{{m}^{-1}}\]for \[\text{H}{{\text{e}}^{\text{+}}}\text{,}\]then its value for \[B{{e}^{3+}}\](isoelectronic of\[H{{e}^{+}},\]) for same transition is

    A)
    \[\frac{x}{4}c{{m}^{-1}}\]            

    B)
           \[x\,c{{m}^{-1}}\]

    C)
    \[4x\,c{{m}^{-1}}\]         

    D)
           \[16x\,c{{m}^{-1}}\]

    View Answer play_arrow
  • question_answer95) \[{{\text{H}}_{\text{2}}}{{\text{O}}_{\text{2}}}\]oxidizes \[\text{Mn}{{\text{O}}_{\text{2}}}\]is \[\text{MnO}_{4}^{-}\]in basic  medium, \[{{\text{H}}_{\text{2}}}{{\text{O}}_{\text{2}}}\]and \[\text{Mn}{{\text{O}}_{\text{2}}}\]react in the molar ratio of

    A)
    1 : 1     

    B)
                           2 : 1       

    C)
           2 : 3       

    D)
           3 : 2

    View Answer play_arrow
  • question_answer96) 25 mL of an aqueous solution of \[\text{KCl}\]was found to require 20 mL of \[\text{1}\,\text{M}\,\text{AgNO}{{}_{\text{3}}}\]solution when titrated using a \[{{\text{K}}_{\text{2}}}\text{Cr}{{\text{O}}_{\text{4}}}\]as indicator. Depression in freezing point of \[\text{KCl}\]solution with 100% ionization will be [\[{{K}_{F}}={{2.0}^{o}}mo{{l}^{-1}}kg,\]molarity = molality]

    A)
    \[{{3.2}^{o}}\]                  

    B)
           \[{{1.6}^{o}}\]                  

    C)
    \[{{0.8}^{o}}\]                  

    D)
           \[{{5.0}^{o}}\]

    View Answer play_arrow
  • question_answer97) Out of the following, select Lux-Flood Acid

    A)
    \[C{{O}_{2}}\]                                   

    B)
    \[B{{F}_{3}}\]                    

    C)
    \[{{H}^{+}}\]                     

    D)
           \[Al{{(C{{H}_{3}})}_{3}}\]

    View Answer play_arrow
  • question_answer98) On adding \[\text{AgN}{{\text{O}}_{\text{3}}}\]solution into KI solution, a negatively charged colloidal sol. Is obtained when they are in

    A)
    \[100\,mL\,of0.1\text{ }M\,AgN{{O}_{3}}\text{ }+\text{ }100\,mL\,of0.1\text{ }M\,Kl\]

    B)
    \[100\,mL\,of0.1\text{ }M\,AgN{{O}_{3}}+100\,mL\,of0.2\text{ m}\,Kl\]

    C)
    \[100\,mL\,of0.2\text{ }M\,AgN{{O}_{3}}+100\,mL\,of0.1\text{ M}\,Kl\]

    D)
    \[100\,mL\,of0.15\text{ }M\,AgN{{O}_{3}}+100\,mL\,of0.15\text{ M}\,Kl\]

    View Answer play_arrow
  • question_answer99) Graph between \[\log \left( \frac{x}{m} \right)\]and log p is a straight line at angle \[{{45}^{o}}\] with intercept OA as shown. Hence, \[\left( \frac{x}{m} \right)\]at a pressure of 0.2 atm is

    A)
    0.4                    

    B)
           0.6                         

    C)
    0.8                 

    D)
    0.2

    View Answer play_arrow
  • question_answer100) From the following reactions at 298 K,
    (A)\[Ca{{C}_{2}}(s)+2{{H}_{2}}O(l)\xrightarrow{{}}Ca{{(OH)}_{2}}(s)\] \[+\,{{C}_{2}}{{H}_{2}}(g)D{{H}^{o}}(kJ\,mo{{l}^{-1}})-127.9\]
    (B) \[Ca(s)+\frac{1}{2}{{O}_{2}}(g)\xrightarrow{{}}CaO(s)-635.1\]
    (C) \[CaO(s)+{{H}_{2}}O(I)\xrightarrow{{}}Ca{{(OH)}_{2}}(s)-65.2\]
    (D) \[C(s)+{{O}_{2}}(s)\xrightarrow{{}}C{{O}_{2}}(s)-393.5\]
    (E) \[{{C}_{2}}{{H}_{2}}(g)+\frac{5}{2}{{O}_{2}}(g)\xrightarrow{{}}2C{{O}_{2}}(g)\] \[+\,{{H}_{2}}O(e)-1299.58\]
    Calculate the heat of formation of \[Ca{{C}_{2}}(s)\]at 298 K.

    A)
    \[-\text{ }59.82\text{ }KJ\text{ }mo{{l}^{-1}}\]     

    B)
    \[~+\text{ }59.82\text{ }KJ\text{ }mo{{l}^{-1}}\]

    C)
    \[-190.22\text{ }KJ\text{ }mo{{l}^{-1}}\]

    D)
    \[+\,190.22\,KJ\,mo{{l}^{-1}}\]

    View Answer play_arrow
  • question_answer101) If\[{{x}_{r}}=\cos \left( \frac{\pi }{{{3}^{r}}} \right)-i\sin \left( \frac{\pi }{{{3}^{r}}} \right),\](where\[i=\sqrt{-1}\]),  then the value of\[{{x}_{1}}.{{x}_{2}}...\infty ,\]is

    A)
     1                                            

    B)
     \[-1\]                   

    C)
     \[-\,i\]                 

    D)
            \[\,i\]

    View Answer play_arrow
  • question_answer102) Number of identical terms in the sequence 2, 5, 8, 11,... upto 100 terms and 3, 5, 7, 9, 11,... upto 100 terms are

    A)
     17                         

    B)
     33         

    C)
            50         

    D)
            147

    View Answer play_arrow
  • question_answer103) If \[1+\lambda +{{\lambda }^{2}}+...+{{\lambda }^{n}}=(1+\lambda )+(1+{{\lambda }^{2}})(1+{{\lambda }^{4}})\]\[(1+{{\lambda }^{8}})(1+{{\lambda }^{16}}),\] then the value of n is (where,\[n\in N\])

    A)
     32                         

    B)
     16                         

    C)
            31                  

    D)
            15

    View Answer play_arrow
  • question_answer104) If one root of the equation \[{{x}^{2}}-\lambda x+12=0\]is even prime while \[{{x}^{2}}+\lambda x+\mu =0\]has equal roots, then \[\mu \] is equal to

    A)
     8                                            

    B)
     16

    C)
     24                  

    D)
                                            32

    View Answer play_arrow
  • question_answer105) If \[\alpha ,\]and \[\beta \]are the roots of \[a{{x}^{2}}+c=bx,\]then the equation \[{{(a+cy)}^{2}}={{b}^{2}}y\]in \[y\]has the roots

    A)
     \[{{\alpha }^{-1}},{{\beta }^{-1}}\]                          

    B)
     \[{{\alpha }^{2}},{{\beta }^{2}}\]             

    C)
            \[\alpha {{\beta }^{-1}},{{\alpha }^{-1}}\beta \]               

    D)
     \[{{\alpha }^{-2}},{{\beta }^{-2}}\]

    View Answer play_arrow
  • question_answer106) If \[{{\,}^{n}}{{C}_{r-1}}=10,{{\,}^{n}}{{C}_{r}}=45\] and \[{{\,}^{n}}{{C}_{r+1}}=120,\] then r equals to

    A)
     1                       

    B)
     2                            

    C)
     3                     

    D)
     4

    View Answer play_arrow
  • question_answer107) The number of six-digit numbers that can be formed from the digits 1, 2, 3, 4, 5, 6 and 7, so that digits do not repeat and the terminal digits are even is

    A)
     144                

    B)
            72                         

    C)
     288                

    D)
            720

    View Answer play_arrow
  • question_answer108) The coefficient of \[{{x}^{20}}\]in the expansion of \[{{(1+{{x}^{2}})}^{40}}.{{\left( {{x}^{2}}+2+\frac{1}{{{x}^{2}}} \right)}^{-5}}\]is

    A)
     \[{{\,}^{20}}{{C}_{10}}\]              

    B)
           \[{{\,}^{30}}{{C}_{25}}\]               

    C)
            1                            

    D)
            0

    View Answer play_arrow
  • question_answer109) If \[\Delta =\left| \begin{matrix}    1 & \sin \theta  & 1  \\    -\sin \theta  & 1 & \sin \theta   \\    -1 & -\sin \theta  & 1  \\ \end{matrix} \right|;0\le \theta <2\pi ,\] then

    A)
    \[\Delta =0\]     

    B)
            \[\Delta \in (0,\infty )\]               

    C)
            \[\Delta \in [-1,2]\]       

    D)
            \[\Delta \in [2,4]\]

    View Answer play_arrow
  • question_answer110) If \[A=\left[ \begin{matrix}    10  \\    -17  \\ \end{matrix} \right]\]and \[{{A}^{2}}=8A+K{{I}_{2}},\]then K is equal to

    A)
     -1                          

    B)
            1                            

    C)
     -7                          

    D)
            7

    View Answer play_arrow
  • question_answer111) If \[x\in \left( \frac{3\pi }{2},2\pi  \right),\]then the value of the expression \[{{\sin }^{-1}}[\cos \{{{\cos }^{-1}}(\cos x)\}+{{\sin }^{-1}}(\sin x)],\]is

    A)
     \[-\frac{\pi }{2}\]                            

    B)
            \[\frac{\pi }{2}\]                             

    C)
     0                            

    D)
            \[\pi \]

    View Answer play_arrow
  • question_answer112) The value of \[a\] for which \[a{{x}^{2}}+{{\sin }^{-1}}({{x}^{2}}-2x+2)+{{\cos }^{-1}}\]\[({{x}^{2}}-2x+2)=0\] has a real solution, is

    A)
    \[-\frac{2}{\pi }\]                             

    B)
            \[\frac{2}{\pi }\]                             

    C)
     \[-\frac{\pi }{2}\]                            

    D)
            \[\frac{\pi }{2}\]

    View Answer play_arrow
  • question_answer113) If \[\sin x+\cos x=\sqrt{\left( y+\frac{1}{y} \right)},x\in [0,\pi ],\]then

    A)
     \[x=\frac{\pi }{4},y=1\]

    B)
            \[y=0\]

    C)
            \[y=2\]

    D)
            \[x=\frac{3\pi }{4}\]

    View Answer play_arrow
  • question_answer114) If \[n\] be a positive integer such that \[\sin \left( \frac{\pi }{2n} \right)+\cos \left( \frac{\pi }{2n} \right)=\frac{\sqrt{n}}{2},\]then

    A)
    \[n=6\]               

    B)
           \[n=2\]

    C)
    \[n=1\]               

    D)
            \[n=3,4,5\]

    View Answer play_arrow
  • question_answer115) If \[\tan x.\tan y=a\]and \[x+y=\frac{\pi }{6},\]then tan \[x\]and \[\tan \,y\]satisfy the equation

    A)
     \[{{x}^{2}}-\sqrt{3}(1-a)x+a=0\]

    B)
     \[\sqrt{3}{{x}^{2}}-(1-a)x+a\sqrt{3}=0\]

    C)
     \[{{x}^{2}}+\sqrt{3}(1+a)x-a=0\]

    D)
     \[\sqrt{3}{{x}^{2}}+(1+a)x-a\sqrt{3}=0\]

    View Answer play_arrow
  • question_answer116) The angle of elevation of the top of a tower at a point on the ground is \[\text{3}{{\text{0}}^{\text{o}}}\text{.}\] If on walking 20 m toward the tower, the angle of elevation becomes \[\text{6}{{\text{0}}^{\text{o}}},\] then the height of the tower is

    A)
     \[10\,m\]           

    B)
            \[\frac{10}{\sqrt{3}}m\]              

    C)
            \[10\sqrt{3}\,m\]           

    D)
            None of these

    View Answer play_arrow
  • question_answer117) The centroid of the triangle is \[(3,3)\] and the orthocentre is \[(-3,5),\] then its circumcentre is

    A)
     (0, 4)               

    B)
            (0, 8)    

    C)
            (6, 2)               

    D)
            (6,-2)

    View Answer play_arrow
  • question_answer118) Point \[R(h,k)\]divides a line segment between the axes in the ratio 1 : 2. Find equation of the line.

    A)
     \[2kx+hy=3hk\]

    B)
     \[2kx+hy=2hk\]

    C)
     \[2kx-hy=3hk\]

    D)
     None of the above

    View Answer play_arrow
  • question_answer119) If the slope of one of the lines represented by \[a{{x}^{2}}+2hxy+b{{y}^{2}}=0\]be the square of the other, then

    A)
     \[=ab(a+b)-6abh+8{{h}^{3}}=0\]

    B)
     \[{{a}^{2}}b+ab+6abh+8h=0\]

    C)
     \[{{a}^{2}}b+a{{b}^{2}}-3abh+8{{h}^{3}}=0\]

    D)
     \[{{a}^{2}}b+a{{b}^{2}}-6abh-8{{h}^{3}}=0\]

    View Answer play_arrow
  • question_answer120) The    line   \[(x-2)\cos \beta +(y-2)\sin \theta =1\] touches a circle for all value of \[\theta ,\] then the equation of circle is

    A)
     \[{{x}^{2}}+{{y}^{2}}-4x-4y+7=0\]

    B)
     \[{{x}^{2}}+{{y}^{2}}+4x+4y+7=0\]

    C)
     \[{{x}^{2}}+{{y}^{2}}-4x-4y-7=0\]

    D)
     None of the above

    View Answer play_arrow
  • question_answer121) If \[(-3,2)\] lies on the circle \[{{x}^{2}}+{{y}^{2}}+2gx+2fy+c=0\]which is concentric with the circle\[{{x}^{2}}+{{y}^{2}}+6x+8y-5=0,\] then C is equal to

    A)
     11                         

    B)
           \[-11\]                 

    C)
     24                         

    D)
            100

    View Answer play_arrow
  • question_answer122) The angle between the tangents drawn from the origin to the parabola \[{{y}^{2}}=4a(x-a)\]is

    A)
     0                            

    B)
            \[\frac{\pi }{2}\]                             

    C)
     \[\frac{\pi }{4}\]                             

    D)
            \[\frac{\pi }{6}\]

    View Answer play_arrow
  • question_answer123) If focii of \[\frac{{{x}^{2}}}{{{a}^{2}}}-\frac{{{y}^{2}}}{{{b}^{2}}}=1\] coincide with the foci of \[\frac{{{x}^{2}}}{25}+\frac{{{y}^{2}}}{9}=1\]and eccentricity of the  hyperbola is 2, then

    A)
     \[{{a}^{2}}+{{b}^{2}}=14\]

    B)
     there is a director circle of the hyperbola

    C)
     centre of the director circle is (0, 0)

    D)
     length of latusrectum of the hyperbola is

    View Answer play_arrow
  • question_answer124) \[\frac{\frac{1}{2!}+\frac{1}{4!}+\frac{1}{6!}+...\infty }{1+\frac{1}{3!}+\frac{1}{5!}+\frac{1}{7!}+...\infty }\]is equal to

    A)
     \[\frac{e+1}{e-1}\]        

    B)
            \[\frac{e-1}{e+1}\]        

    C)
            \[\frac{{{e}^{2}}+1}{{{e}^{2}}-1}\]          

    D)
            \[\frac{{{e}^{2}}-1}{{{e}^{2}}+1}\]

    View Answer play_arrow
  • question_answer125) If n is even, then in the expansion of  \[{{\left( 1+\frac{{{x}^{2}}}{2!}+\frac{{{x}^{4}}}{4!}+... \right)}^{2}},\]then the coefficient of \[{{x}^{n}}\]is

    A)
     \[\frac{{{2}^{n}}}{n!}\]                

    B)
            \[\frac{{{2}^{n}}-2}{n!}\]            

    C)
     \[\frac{{{2}^{n-1}}-1}{n!}\]        

    D)
     \[\frac{{{2}^{n-1}}}{n!}\]

    View Answer play_arrow
  • question_answer126) The angle between the line \[\frac{x+1}{2!}=\frac{y}{3}=\frac{z-3}{6}\]and the plane \[10x+2y-11z=3\] is

    A)
     \[\frac{\pi }{2}\]                             

    B)
            \[\frac{\pi }{4}\]                             

    C)
     \[\frac{\pi }{6}\]                             

    D)
            \[{{\sin }^{-1}}\left( \frac{8}{21} \right)\]

    View Answer play_arrow
  • question_answer127) The area of the triangle whose vertices are A (1, 2, 3), \[B(2,-1,1)\]and \[C(1,2,-4)\]is

    A)
     \[7\sqrt{10}\,sq\]units

    B)
            \[\frac{1}{2}\sqrt{10}\,\text{sq}\,\text{units}\]

    C)
            \[\frac{7}{2}\sqrt{10}\,\text{sq}\,\text{units}\]

    D)
            None of these

    View Answer play_arrow
  • question_answer128) If a and b are the vectors determined by two adjacent sides of regular hexagon, then vector EO is

    A)
    \[(a+b)\]           

    B)
           \[(a-b)\]              

    C)
            2 a                        

    D)
            2b

    View Answer play_arrow
  • question_answer129) If \[a=i+j+k,\]  \[b=4i+3j+4k\]and \[c=i+\alpha j+\beta k\]are linearly dependent  vectors and \[|c|=\sqrt{3},\] then the values of \[\alpha \] and \[\beta \]are respectively

    A)
    \[\pm \,1,1\]                     

    B)
            \[\pm \,2,1\]    

    C)
            \[0,\pm \,1\]    

    D)
            None of these

    View Answer play_arrow
  • question_answer130) Let \[u=i+j,v=i-j\]and \[w=i+2j+3k\]. If \[\hat{n}\] is a unit vector such that \[u.\hat{n}=0\]and \[u.\hat{n}=0,\]then \[|w.\hat{n}|\]is equal to

    A)
     3                            

    B)
     0                            

    C)
     1                     

    D)
     2

    View Answer play_arrow
  • question_answer131) If \[g(x)=1+\sqrt{x}\]and \[f\{g(x)\}\]\[=3+2\sqrt{x}+x,\] then \[f(x)\]is equal to

    A)
    \[1+2{{x}^{2}}\]           

    B)
           \[~2+{{x}^{2}}\]               

    C)
           \[1+x\]                

    D)
           \[~2+x\]

    View Answer play_arrow
  • question_answer132) Domain of \[f(x)=y=\sqrt{{{\log }_{3}}\{\cos (\sin x)\}}\]is

    A)
     \[\left\{ \frac{n\pi }{2}:n\in l \right\}\]

    B)
            \[\{2n\pi :n\in l\}\]

    C)
     \[\{n\pi :n\in l\}\]          

    D)
            None of the above

    View Answer play_arrow
  • question_answer133) \[\underset{x\to 1}{\mathop{\lim }}\,({{\log }_{2}}2x{{\log }_{x}}5)\]is equal to

    A)
     \[{{\log }_{2}}5\]            

    B)
            \[_{e}{{\log }_{2}}5\]    

    C)
            e                           

    D)
            0

    View Answer play_arrow
  • question_answer134) A function is defined as follows             \[f(x)=\left\{ \begin{matrix}    1, & \text{when}-\infty <x<0  \\    1+\sin x, & \text{when}0\le x<\frac{\pi }{2}  \\    2+{{\left( x-\frac{\pi }{2} \right)}^{2}}, & \text{when}\frac{\pi }{2}\le x<\infty   \\ \end{matrix} \right.\] continuity of \[f(x)\]is

    A)
     \[f(x)\]is continuous at \[x=\frac{\pi }{2}\]

    B)
    (b \[f(x)\]is continuous at\[x=0\]                  

    C)
     \[f(x)\] is discontinuous at \[x=0\]

    D)
     \[f(x)\] is continuous over the whole real number

    View Answer play_arrow
  • question_answer135) If \[f(x)=\sqrt{1-\sqrt{1-{{x}^{2}}}},\]then \[f(x)\]is

    A)
     continuous on \[[-1,1]\]

    B)
     differentiable on \[(-1,0)\cup (0,1)\]

    C)
     Both (a) and (b)

    D)
     None of the above

    View Answer play_arrow
  • question_answer136) If \[x=\sqrt{{{a}^{{{\sin }^{-1}}}}t,}\]and\[y=\sqrt{{{a}^{{{\cos }^{-1}}}}t,}\] then the value of \[\frac{dy}{dx}\]is

    A)
     \[\frac{y}{x}\]                  

    B)
            \[\frac{x}{y}\]

    C)
     \[\frac{-y}{x}\]                

    D)
            \[\frac{-x}{y}\]

    View Answer play_arrow
  • question_answer137) If f and g be differentiable function satisfying \[g(a)=2,g(a)=b\]and\[fog=I\] (identity function), then \[f(b)\]is equal to

    A)
     \[\frac{1}{2}\]                  

    B)
            2                            

    C)
     \[\frac{2}{3}\]                  

    D)
            None of these

    View Answer play_arrow
  • question_answer138) If \[x=a{{t}^{2}}\]and \[y=2at,\]then \[\frac{{{d}^{2}}y}{d{{x}^{2}}}\]at \[t=2,\]is

    A)
     \[\frac{-1}{16a}\]           

    B)
            \[\frac{1}{16a}\]             

    C)
            \[\frac{1}{16}\]               

    D)
            \[\frac{1}{a}\]

    View Answer play_arrow
  • question_answer139) The points at which the tangent to the curve  \[y={{x}^{3}}-3{{x}^{2}}-9x+7\]is parallel to the \[x-\]axis are

    A)
     \[(30-20)\]and\[(-1,12)\]

    B)
     (3, 20) and (1, 12)

    C)
    \[(1,-10)\]and (2, 6)   

    D)
     None of these

    View Answer play_arrow
  • question_answer140) \[\int_{{}}^{{}}{\frac{dx}{{{\sin }^{2}}x.{{\cos }^{2}}x}}\]is equal to

    A)
     \[\tan x+d\cot x+C\]    

    B)
     \[\tan x-\cot x+C\]

    C)
     \[\tan x.\cot x+C\]

    D)
     \[\tan x-\cot 2x+C\]

    View Answer play_arrow
  • question_answer141) \[\int_{{}}^{{}}{\frac{{{e}^{2x}}-1}{{{e}^{2x}}+1}}dx\]is equal to

    A)
    \[\frac{{{e}^{x}}+{{e}^{-x}}}{{{e}^{x}}-{{e}^{-x}}}+C\]    

    B)
            \[\log \frac{{{e}^{x}}+{{e}^{-x}}}{{{e}^{x}}-{{e}^{-x}}}+C\]          

    C)
     \[\log |{{e}^{x}}+{{e}^{-x}}|+C\]            

    D)
     \[\log |{{e}^{x}}-{{e}^{-x}}|+C\]

    View Answer play_arrow
  • question_answer142) The value of \[\int_{0}^{1}{{{\tan }^{-1}}\left( \frac{2x-1}{1+x-{{x}^{2}}} \right)dx}\]is

    A)
     1                            

    B)
            0                            

    C)
     \[-1\]                   

    D)
            \[\frac{\pi }{4}\]

    View Answer play_arrow
  • question_answer143) If \[\int_{\log 2}^{\pi }{\frac{1}{\sqrt{{{e}^{x}}-1}}}dx=\frac{\pi }{6},\] then the value of \[x\] is

    A)
     log 2               

    B)
            log 3     

    C)
            log 4               

    D)
            None of these

    View Answer play_arrow
  • question_answer144) The area bounded by the curve\[y=\sin x\]between \[x=0\]and \[x=2\pi \]is

    A)
     1 sq unit             

    B)
            2 sq units           

    C)
            4 sq units            

    D)
             8 sq units

    View Answer play_arrow
  • question_answer145) If the area above \[x-\]axis bounded by the n curves \[y={{2}^{Kx}},x=0\]and\[x=2\] is\[\frac{3}{\log 2},\]then  the value of K is

    A)
     \[\frac{1}{2}\]                  

    B)
                            1                            

    C)
            \[-1\]                   

    D)
                            2

    View Answer play_arrow
  • question_answer146) The solution of the differential equation \[{{\sec }^{2}}x.\tan ydx+{{\sec }^{2}}y.\tan x\,dy=0\]is

    A)
    \[\tan x.\cot y=C\]

    B)
           \[\cot x.tany=C\]

    C)
            \[\tan x.tany=C\]

    D)
            \[\sin x.\cos y=C\]

    View Answer play_arrow
  • question_answer147) Two cards are drawn at random from a pack of 52 cards. The probability of getting at least a spade and an ace, is

    A)
     \[\frac{1}{34}\]               

    B)
           \[\frac{8}{221}\]                              

    C)
     \[\frac{1}{26}\]               

    D)
            \[\frac{2}{51}\]

    View Answer play_arrow
  • question_answer148) The relation R defined in the set {1, 2, 3, 4, 5, 6} as R = {(a, b): b = a + 1} is

    A)
     reflexive            

    B)
            symmetric         

    C)
     transitive       

    D)
     None of these

    View Answer play_arrow
  • question_answer149) The proposition \[\tilde{\ }(p\Leftrightarrow q)\]is equivalent to

    A)
    \[(p\vee \tilde{\ }q)\wedge (q\wedge \tilde{\ }p)\]       

    B)
     \[(p\wedge \tilde{\ }q)\vee (q\wedge \tilde{\ }p)\]

    C)
     \[(p\wedge \tilde{\ }q)\wedge (q\wedge \tilde{\ }p)\]

    D)
     None of the above

    View Answer play_arrow
  • question_answer150) When two equal forces are inclined at an angle \[2\alpha ,\] their resultant is twice as great as when they act at an angle \[2\beta ,\] then

    A)
     \[\cos \alpha =2\cos \beta \]

    B)
           \[\cos \alpha =2\sin \beta \]

    C)
            \[\cos \beta =2\cos \alpha \]

    D)
            \[\sin \beta =\cos \alpha \]

    View Answer play_arrow

Study Package

   


You need to login to perform this action.
You will be redirected in 3 sec spinner